ChaseDream
搜索
返回列表 发新帖
查看: 4806|回复: 7
打印 上一主题 下一主题

官方og逻辑两版对比问题,求助,急求

[复制链接]
跳转到指定楼层
楼主
发表于 2010-11-13 10:43:07 | 只看该作者 回帖奖励 |倒序浏览 |阅读模式
7.A researcher discovered that people who have low levels of immune’ system activity tend to score much lower on tests of mental health than do people with normal or high immune’ system activityThe researcher concluded from this experiment that the immune system protects against mental illness as well as against physical disease

The researcher's conclusion depends on which of the following assumptions?

(A)High immunesystem activity protects against mental illness better than normal immune, system activity does

(B)Mental illness is similar to physical disease in its effects on body systems.

(C)People with high immune-system activity cannot develop mental illness·

(D)Mental illness does not cause people's immune-system activity to decrease.

(E)Psychological treatment of mental illness is not as effective as is medical treatment
这题想问XDJJMM
这个题目属于假设题中的事实原因类吗?如果是的话,能指一下哪块是原因/事实?
还有这道题的B选项和下面这道题的E选项完全雷同,我的问题是为啥下面这个就能选E,而我上面这个,就不能选B。我的思路是下面这个选项对的是因为书与软件近似所以都应该享有同样待遇。
那B不也是同一个道理吗?B:精神病与一般身体疾病在人体上产生相同效果,所以免疫系统反应也相同。为啥不能选B???
OG12--58
58. Even though most universities retain the royalties from
faculty members’ inventions, the faculty members
retain the royalties from books and articles they write.
Therefore, faculty members should retain the royalties
from the educational computer software they develop.
The conclusion above would be more reasonably
drawn if which of the following were inserted into the
argument as an additional premise?
正确盗案e选项完全类同:(E) In terms of the criteria used to award royalties,
educational software programs are more nearly
comparable to books and articles than to
inventions.
我的问题是上面那个。
收藏收藏 收藏收藏
沙发
发表于 2010-11-13 12:55:32 | 只看该作者
A researcher discovered that people who have low levels of immune’ system activity tend to score much lower on tests of mental health than do people with normal or high immune’ system activity.The researcher concluded from this experiment that the immune system protects against mental illness as well as against physical disease.

The researcher's conclusion depends on which of the following assumptions?

(A)High immune.system activity protects against mental illness better than normal immune, system activity does.

(B)Mental illness is similar to physical disease in its effects on body systems.

(C)People with high immune-system activity cannot develop mental illness·

(D)Mental illness does not cause people's immune-system activity to decrease.

(E)Psychological treatment of mental illness is not as effective as is medical treatment

- - - - -

A typical necessary assumption question.

Fact: Correlation of poor immune system and poor mental health.  These two events happen together.
Conclusion:  immune system protects against mental illness as well as against physical disease. One event causes the other event.

However, when events A and B happen together, the possibilities are: 1) A causes B; 2) B causes A; 3) a third event C causes both A and B. When the argument concluded that presence of poor immune system CAUSES poor mental health, the argument also assumes that poor mental health does not cause poor immune system.  

And answer (D) speaks just that.  Right answer.

As to answer (B) Mental illness is similar to physical disease in its effects on body systems, even if mental illness is NOT similar to physical disease in its effects on body systems, the argument still holds as the author already commits to the assumption that the immune system failure causes the poor mental health.  Hence, answer choice (B) has no bearing on the credibility of the conclusion drawn.  Remember, we are looking for a NECESSARY assumption, which is required by the argument.
板凳
发表于 2010-11-13 13:00:01 | 只看该作者
Even though most universities retain the royalties from
faculty members’ inventions, the faculty members
retain the royalties from books and articles they write.
Therefore, faculty members should retain the royalties
from the educational computer software they develop.
The conclusion above would be more reasonably
drawn if which of the following were inserted into the
argument as an additional premise?
正确盗案e选项完全类同:(E) In terms of the criteria used to award royalties,
educational software programs are more nearly
comparable to books and articles than to
inventions.

- - - - -

This is a strengthen question.  Basically there are two ways to categorize a product.  The answer choice adds more weight to one of the choice.
地板
 楼主| 发表于 2010-11-19 21:06:22 | 只看该作者
哦,很清晰,谢谢楼上
5#
发表于 2011-8-3 23:32:11 | 只看该作者
COOL explanation.
THKS!
6#
发表于 2011-10-19 05:43:32 | 只看该作者
A researcher discovered that people who have low levels of immune’ system activity tend to score much lower on tests of mental health than do people with normal or high immune’ system activity.The researcher concluded from this experiment that the immune system protects against mental illness as well as against physical disease.

The researcher's conclusion depends on which of the following assumptions?

(A)High immune.system activity protects against mental illness better than normal immune, system activity does.

(B)Mental illness is similar to physical disease in its effects on body systems.

(C)People with high immune-system activity cannot develop mental illness·

(D)Mental illness does not cause people's immune-system activity to decrease.

(E)Psychological treatment of mental illness is not as effective as is medical treatment

- - - - -

A typical necessary assumption question.

Fact: Correlation of poor immune system and poor mental health.  These two events happen together.
Conclusion:  immune system protects against mental illness as well as against physical disease. One event causes the other event.

However, when events A and B happen together, the possibilities are: 1) A causes B; 2) B causes A; 3) a third event C causes both A and B. When the argument concluded that presence of poor immune system CAUSES poor mental health, the argument also assumes that poor mental health does not cause poor immune system.  

And answer (D) speaks just that.  Right answer.

请问作者什么时候推出的这个结论?在结论里作者没有说  poor immune system CAUSES poor mental health  啊,只是说免疫系统能防止得精神疾病,也防止身体疾病。 这个CAUSES的关系是怎么的得出来的?
7#
发表于 2012-1-6 16:49:55 | 只看该作者
Premise:
研究人员发现到 与免疫系统正常或较高的人员相较下 免疫系统低下的人(A)有着较低的心理健康测试值(B)的倾向
两个事件同时存在,有可能1)A因B果 2)B因A果 3)C因

Conclusion:
免疫系统不但能防止得精神疾病,也能防止身体疾病
由此看来 免疫系统的好坏是因 有无身体疾病是果 即1)A因B果的情况

D)明确了不可能B因A果,所以是好答案
8#
发表于 2012-6-28 00:30:45 | 只看该作者
Premise:
研究人员发现到 与免疫系统正常或较高的人员相较下 免疫系统低下的人(A)有着较低的心理健康测试值(B)的倾向
两个事件同时存在,有可能1)A因B果 2)B因A果 3)C因

Conclusion:
免疫系统不但能防止得精神疾病,也能防止身体疾病
由此看来 免疫系统的好坏是因 有无身体疾病是果 即1)A因B果的情况

D)明确了不可能B因A果,所以是好答案
-- by 会员 wildmantomba (2012/1/6 16:49:55)



多谢,终于看到一个明白的答案了~
您需要登录后才可以回帖 登录 | 立即注册

Mark一下! 看一下! 顶楼主! 感谢分享! 快速回复:

手机版|ChaseDream|GMT+8, 2025-2-18 09:52
京公网安备11010202008513号 京ICP证101109号 京ICP备12012021号

ChaseDream 论坛

© 2003-2023 ChaseDream.com. All Rights Reserved.

返回顶部